irini101
Thanks Received: 1
Forum Guests
 
Posts: 49
Joined: August 30th, 2011
 
 
 

Q1 - Studies have shown that treating

by irini101 Tue Oct 25, 2011 6:56 pm

I choose A by POE. To weaken drugs used on human by test on animal reminds me of common scenario in LR: weaken/ strengthen human drug by animal test.

I have seen comments saying testing the drug on animal is irrelevant to the effect it applied on human, which obviously conflicts (A) in this question.

Could any one confirm whether the result of testing a drug on animal does weaken / strengthen its effectiveness on human?

Thanks a lot!
 
extraordinary.kye
Thanks Received: 5
Forum Guests
 
Posts: 12
Joined: November 17th, 2011
 
 
 

Re: Q1 - Studies have shown that treating

by extraordinary.kye Sat Nov 19, 2011 8:44 am

When i was solving this problem...

I choose answer choice (A) because if X could produce a bunch of bad side effects, this might put into question whether we should perfer it over Y.

I think dealing with laboratory animals were just small minor issue to consider, thought focusing on "produced harmful side effects" was making the correct choice.

Hope that helps :mrgreen:
 
eunjung.shin
Thanks Received: 2
Jackie Chiles
Jackie Chiles
 
Posts: 40
Joined: December 08th, 2011
 
 
 

Re: Q1 - Studies have shown that treating

by eunjung.shin Thu Jun 07, 2012 9:30 pm

Can't you weaken by giving an alternative choice? E says Z is better than X. So wouldn't that weaken the conclusion which says X should be preferred to Y?

Confused....
 
sumukh09
Thanks Received: 139
Atticus Finch
Atticus Finch
 
Posts: 327
Joined: June 03rd, 2012
 
 
trophy
Most Thanked
trophy
First Responder
 

Re: Q1 - Studies have shown that treating

by sumukh09 Sun Mar 03, 2013 1:30 am

eunjung.shin Wrote:Can't you weaken by giving an alternative choice? E says Z is better than X. So wouldn't that weaken the conclusion which says X should be preferred to Y?

Confused....


I picked E for this reason as well but I think it's wrong because the argument is only about treatment X and treatment Y so other treatments are irrelevant and need not be considered.

I passed over A because I wasn't sure how effects on laboratory animals were relevant and so I deemed the answer choice out of scope since the argument was specifically about illnesses to humans.
 
niwallace
Thanks Received: 0
Vinny Gambini
Vinny Gambini
 
Posts: 6
Joined: August 21st, 2012
 
 
 

Re: Q1 - Studies have shown that treating

by niwallace Thu May 16, 2013 3:08 pm

Can we get some confirmation here?
User avatar
 
daniel
Thanks Received: 0
Elle Woods
Elle Woods
 
Posts: 62
Joined: July 31st, 2012
Location: Lancaster, CA
 
 
 

Re: Q1 - Studies have shown that treating

by daniel Tue Jul 30, 2013 6:50 pm

We're asked to select an answer choice that weakens the argument. This type of question is part of the Assumption Family, so we should start by identifying the argument core.

P1: X produces same beneficial changes as Y for same illnesses
+
P2: X is quicker and less expensive than Y
==========================
Conclusion: X should be preferred to Y

The argument fails to consider whether there are any reasons not to use X. Maybe the condition being treated improves, but X causes your hair to fall out, but Y does not cause your hair to fall out? Maybe X is cheaper because the company that manufactures it has a history of poor quality control, which in some cases has resulted in poisoning or other complications for patients who were prescribed X.

(A) raises the possibility of harmful side effects in X. This lines up quite well with the gap noted above. Keep for now.
(B) out of scope. We don't care about other illnesses; the argument is very careful to refer to the same illnesses being treated by both X and Y.
(C) irrelevant. The premise states that X is cheaper than Y, and that's all that matters.
(D) out of scope. It doesn't matter whether doctors prescribe one treatment more often than the other, as this is not part of the argument core.
(E) out of scope. Whether or not Z is quicker and less expensive than Y does not weaken the conclusion, which states a preference for X relative to Y. Even if Z is preferred over X, it could still be true that X should be preferred over Y.

(A) is the correct answer. As with other "Weaken" questions, we need only to demonstrate that an assumption made in the argument is potentially not valid. (A) does that by raising a potential issue with respect to side effects. Although it doesn't destroy the argument (maybe the side effects do not manifest in human subjects with the prescribed dosages), it does weaken the argument by attacking an assumption.
 
roflcoptersoisoi
Thanks Received: 0
Atticus Finch
Atticus Finch
 
Posts: 165
Joined: April 30th, 2015
 
 
 

Re: Q1 - Studies have shown that treating

by roflcoptersoisoi Fri Jul 15, 2016 7:09 pm

P1: Studies show treating certain illnesses with treatment X produces the same beneficial changes in patients' conditions as treating the same illness with treatment Y

P2: Treatment X is less expensive that treatment Y

Conclusion: Treatment X should be preferred to treatment Y in treating those illnesses.

Analysis: Author takes for granted that treatment x does not produce any negative side effects that negate the beneficial changes it produces.

(A) This touches on a potential side effect of treatment X, we anticipated this when we were identifying potential flaws in the author's reasoning, keep for now.
(B) Who cares about other illnesses? We're talking about the illnesses mentioned in the stimulus. If this were true, treatment X could still be a better option than treatment Y for the illnesses mentioned in the stimulus. This doesn't affect the argument in any way, eliminate.
(C) This has absolutely no bearing on the argument. We don't care about how much treatment X or Y used to cost.
(D) For this to weaken the argument we'd have to assume that the frequency with which a treatment is prescribed equates to how preferable it is, but we don't know if this is the case, remember any answer choice that requires conjecture in order to be an acceptable answer choice is wrong.. Eliminate.
(E) Out of scope. We don't care about any other treatments. Even if treatment Z was quicker and less expensive than treatment X, the latter could still be preferable than treatment Y. Remember, we're concerned about treatment X and Y only, STICK TO THE CORE!!


(A) This is clearly the best answer. Despite the fact that it produces the same benefits as treatment Y, it also produces harmful side effects, therefore making it less likely that it should be preferred to treatment Y in regards to the illnesses in question.
 
roflcoptersoisoi
Thanks Received: 0
Atticus Finch
Atticus Finch
 
Posts: 165
Joined: April 30th, 2015
 
 
 

Re: Q1 - Studies have shown that treating

by roflcoptersoisoi Fri Jul 15, 2016 7:14 pm

irini101 Wrote:I choose A by POE. To weaken drugs used on human by test on animal reminds me of common scenario in LR: weaken/ strengthen human drug by animal test.

I have seen comments saying testing the drug on animal is irrelevant to the effect it applied on human, which obviously conflicts (A) in this question.

Could any one confirm whether the result of testing a drug on animal does weaken / strengthen its effectiveness on human?

Thanks a lot!


Not that we're never actually told that the treatments in question are actually for humans, perhaps they are for animals. Careful of making unwarranted assumptions. ;)
 
roflcoptersoisoi
Thanks Received: 0
Atticus Finch
Atticus Finch
 
Posts: 165
Joined: April 30th, 2015
 
 
 

Re: Q1 - Studies have shown that treating

by roflcoptersoisoi Fri Jul 15, 2016 7:26 pm

sumukh09 Wrote:
eunjung.shin Wrote:Can't you weaken by giving an alternative choice? E says Z is better than X. So wouldn't that weaken the conclusion which says X should be preferred to Y?

Confused....


I picked E for this reason as well but I think it's wrong because the argument is only about treatment X and treatment Y so other treatments are irrelevant and need not be considered.

I passed over A because I wasn't sure how effects on laboratory animals were relevant and so I deemed the answer choice out of scope since the argument was specifically about illnesses to humans.



Weakening answer choices are a bit strange, in that unlike assumption questions, subject matter that is not directly mentioned in the core would still affect the argument, and thus should not be immediately dismissed under the pretext of being "out of scope".
The correct answer to many weakening questions are those that present alternative explanations. However this is only the case when the author presents a phenomenon and gives a hypothesis. A competing hypothesis (alternative exaplanation) will thus invariably weaken the argument, however that is not what is occurring here.
 
LeonC641
Thanks Received: 0
Jackie Chiles
Jackie Chiles
 
Posts: 33
Joined: May 20th, 2018
 
 
 

Re: Q1 - Studies have shown that treating

by LeonC641 Thu Oct 29, 2020 5:18 am

I got this one wrong for thinking laboratory animals not relevant. Then, after pondering quite a while, I came to realize why the answer choice is actually relevant: laboratory animal here is used as an object of a preposition to bring up a specific context, i.e. in a lab. Then, it is reasonable to say those animals' were for some research. So, if some research using animals showed treatment X less preferable than Y, it is reasonable to doubt whether treatment X is conclusively more ideal than Y.
 
YurikaC738
Thanks Received: 0
Vinny Gambini
Vinny Gambini
 
Posts: 12
Joined: February 03rd, 2023
 
 
 

Re: Q1 - Studies have shown that treating

by YurikaC738 Fri Mar 24, 2023 11:11 pm

Another reason to eliminate E is, even Z is quicker and less expensive than X, Z can have fewer beneficial changes compared to X.